Is x < y?

This topic has expert replies
Legendary Member
Posts: 2276
Joined: Sat Oct 14, 2017 6:10 am
Followed by:3 members

Is x < y?

by VJesus12 » Tue Mar 27, 2018 2:51 am

Timer

00:00

Your Answer

A

B

C

D

E

Global Stats

Is x < y?

(1) z < y
(2) z < x

The OA is the option E.

If we subtract both statements we will get that x<y. Why is the correct answer E?<i class="em em-disappointed"></i>

User avatar
GMAT Instructor
Posts: 15539
Joined: Tue May 25, 2010 12:04 pm
Location: New York, NY
Thanked: 13060 times
Followed by:1906 members
GMAT Score:790

by GMATGuruNY » Tue Mar 27, 2018 4:39 am

Timer

00:00

Your Answer

A

B

C

D

E

Global Stats

VJesus12 wrote:Is x < y?

(1) z < y
(2) z < x
Statement 1:
No information about x.
INSUFFICIENT.

Statement 2:
No information about y.
INSUFFICIENT.

Statements combined:
Case 1: z=0, y=2 and x=1
In this case, x < y so the answer to the question stem is YES.
Case 2: z=0, y=1 and x=2
In this case, x > y, so the answer to the question stem is NO.
Since the answer is YES in Case 1 but NO in Case 2, INSUFFICIENT.

The correct answer is E.
If we subtract both statements we will get that x<y. Why is the correct answer E?<i class="em em-disappointed"></i>
We cannot subtract inequalities; we can only ADD them.
Private tutor exclusively for the GMAT and GRE, with over 20 years of experience.
Followed here and elsewhere by over 1900 test-takers.
I have worked with students based in the US, Australia, Taiwan, China, Tajikistan, Kuwait, Saudi Arabia -- a long list of countries.
My students have been admitted to HBS, CBS, Tuck, Yale, Stern, Fuqua -- a long list of top programs.

As a tutor, I don't simply teach you how I would approach problems.
I unlock the best way for YOU to solve problems.

For more information, please email me (Mitch Hunt) at [email protected].
Student Review #1
Student Review #2
Student Review #3

GMAT/MBA Expert

User avatar
GMAT Instructor
Posts: 16207
Joined: Mon Dec 08, 2008 6:26 pm
Location: Vancouver, BC
Thanked: 5254 times
Followed by:1268 members
GMAT Score:770

by Brent@GMATPrepNow » Tue Mar 27, 2018 7:07 am

Timer

00:00

Your Answer

A

B

C

D

E

Global Stats

VJesus12 wrote:Is x < y?

(1) z < y
(2) z < x

The OA is the option E.

If we subtract both statements we will get that x<y. Why is the correct answer E?<i class="em em-disappointed"></i>
Don't subtract inequalities
Doing so can yield erroneous statements.

Consider this example:
5 < 6
0 < 9

When we subtract these inequalities , we get 5 - 0 < 6 - 9
Evaluate to get 5 < -3 (not true)

Cheers,
Brent
Brent Hanneson - Creator of GMATPrepNow.com
Image

GMAT/MBA Expert

User avatar
GMAT Instructor
Posts: 7305
Joined: Sat Apr 25, 2015 10:56 am
Location: Los Angeles, CA
Thanked: 43 times
Followed by:29 members

by Scott@TargetTestPrep » Thu Mar 29, 2018 4:48 pm

Timer

00:00

Your Answer

A

B

C

D

E

Global Stats

VJesus12 wrote:Is x < y?

(1) z < y
(2) z < x
We need to determine if x < y

Statement One Alone:

z < y

Since we know nothing about x, statement one alone is not sufficient to answer the question.

Statement Two Alone:

z < x

Since we know nothing about y, statement one alone is not sufficient to answer the question.

Statements One and Two Together:

Although we know that both x and y are greater than z, we still cannot determine whether y is greater than x.

Answer: E

Scott Woodbury-Stewart
Founder and CEO
[email protected]

Image

See why Target Test Prep is rated 5 out of 5 stars on BEAT the GMAT. Read our reviews

ImageImage